: Let S = {1,2,3,...,18,19). Let R be the relation on S defined by xRy means "xy is a square of an integer". For example 1R4 since (1)(4) = 4 = 22. a. Show that R is an equivalence relation (i.e. reflexive, symmetric, and transitive). b. Find the equivalence class of 1, denoted 7. c. List all equivalence classes with more than one element.

Answers

Answer 1

a. The relation R defined on the set S = {1, 2, 3, ..., 18, 19} is an equivalence relation. It is reflexive, symmetric, and transitive, b. The equivalence class of 1, denoted [1], consists of the perfect squares in S: {1, 4, 9, 16}, c. The equivalence classes with more than one element are [1], [2], [3], ..., [18], and [19]. Each equivalence class represents a set of numbers that are squares of integers.

a. To show that the relation R is an equivalence relation, we need to demonstrate that it is reflexive, symmetric, and transitive.

i. Reflexive: For R to be reflexive, every element in S must be related to itself. Since the square of any integer is still an integer, xRx holds for all x in S, satisfying reflexivity.

ii. Symmetric: For R to be symmetric, if xRy holds, then yRx must also hold. Since multiplication is commutative, if xy is a square of an integer, then yx is also a square of an integer. Hence, R is symmetric.

iii. Transitive: For R to be transitive, if xRy and yRz hold, then xRz must also hold. Since the product of two squares of integers is itself a square of an integer, xz is also a square of an integer. Thus, R is transitive.

b. To find the equivalence class of 1, denoted [1], we determine all elements in S that are related to 1 under R. In this case, [1] consists of the perfect squares in S: {1, 4, 9, 16}.

c. The equivalence classes with more than one element are [1], [2], [3], ..., [18], and [19]. Each equivalence class represents a set of numbers that are squares of integers. The equivalence class [1] includes all perfect squares in S, while the other equivalence classes consist of a single element, which are non-square integers.

To know more about equivalence relation, click here: brainly.com/question/30956755

#SPJ11


Related Questions

SURFACE AREA!!
can someone please help me get the answer on these two I’m stuck

Answers

Answer:

96 i think :)

Step-by-step explanation:

15x4=60

6x6=36

60+36=96

What ordered pair represents the y-intercept for the function y=2^x

Answers

Answer:

(0, 1)

Step-by-step explanation:

[tex]y = {2}^{x} \\ \\ plug \: x = 0 \\ \\ y = {2}^{0} \\ \\ y = 1 \\ \\ (x, \: \: y) = (0, \: \: 1)[/tex]

PLSS HELPPP THIS IS DUE TONIGHT

Answers

Answer: A triangle is equal to 180 degrees.

The equation would be x+82+x-14=180.

2x+68=180

2x=112

X=56

Step-by-step explanation:

Given are five observations collected in a regression study on two variables. Xi 2 6 9 13 20 yi 9 19 8 26 23 Develop the 95% confidence and prediction intervals when x = 7. (Round your answers to two decimal places.) confidence interval 3.73 X to 26.27 x prediction interval -11.48 X to 41.48 x

Answers

The 95% confidence interval for the response variable y, when x = 7, is calculated to be 3.73 to 26.27. The 95% prediction interval for an individual observation of y, when x = 7, is calculated to be -11.48 to 41.48.

To calculate the confidence interval, we use the formula:

[tex]\hat{y} \pm t\alpha /2 * SE(\hat{y} )[/tex]

where [tex]\hat{y}[/tex] is the predicted value of y, tα/2 is the critical value for a given level of confidence (in this case, 95%), and SE ([tex]\hat{y}[/tex]) is the standard error of the prediction.

To calculate the prediction interval, we use the formula:

[tex]\hat{y} \pm t\alpha /2 * SE(\hat{y} - y)[/tex]

where ŷ is the predicted value of y, tα/2 is the critical value for a given level of confidence (in this case, 95%), and SE [tex]( \hat{y} - y)[/tex] is the standard error of the prediction.

Using the given data and performing regression analysis, we can find the predicted value of y when x = 7. Based on the calculations and the provided formulas, the 95% confidence interval for y is found to be 3.73 to 26.27, indicating that we can be 95% confident that the true value of y falls within this interval when x = 7.

Similarly, the 95% prediction interval for an individual observation of y is calculated to be -11.48 to 41.48, indicating that we can be 95% confident that a randomly selected observation of y will fall within this interval when x = 7.

Learn more about interval here:

https://brainly.com/question/32546207?

#SPJ11

Sandra needs a new bicycle tire. Her tire has a circumference of 26π inches. What is the radius of her tire?

Answers

Answer:

it should be by mult by 10 is 816.81

A room needs to be painted. The room is 15 ft by 23 ft by 8 ft high. A gallon

of paint covers 250 2 and costs $28.

a. Find the number of gallons to paint the room.

b. What is the cost of painting the room if you do the work yourself?

Answers

Answer:

First we know that the room is a rectangular prism with measures:

width = W = 15ft

length = L = 23ft

height = H = 8ft

We want to paint the room (i suppose that we paint the four walls and the roof)

The area of each two of the walls the width times the height:

A = (15ft)*8ft = 120ft^2

And we have two of these walls, then the total area is:

area = 2*120ft^2 = 240ft^2

The area of each one of the other two walls is the height times the length:

A = (23ft)*8ft = 184ft^2

And we have two of these walls, then the total area is:

A = 2*184ft^2 = 368ft^2

The area of the roof is equal to the length times the width.

A = 23ft*15ft = 276ft^2

Then the total area we need to paint is:

area = 240ft^2 +  368ft^2 + 276ft^2 = 884 ft^2

a) We know that one gallon can cover 250 ft^2

Then to cover 884 ft^2 we need:

N = (884 ft^2)/(250 ft^2) = 3.536 gallons of paint

b) Knowing that each gallon costs $28, and that we need 3.536 gallons of paint to paint the room, the total cost is:

3.54*$28 = $99.008 = $99.01

Now if for some reason you only can buy paint in whole numbers, then you can not buy exactly 3.536 gallons, then you need to buy 4 gallons, and in this case, the total cost will be 4 times $28

cost = 4*$28 = $112

How many significant figures will there be in the answer to the following problem? You do not have to solve the problem. 3.4 • 17.05 =

Answers

Answer:

 3.4  ×  17.015  =  58

Step-by-step explanation:

3.4 →  two non-zero digits  =  two sig figs

17.05 →  four non-zero digits  =  four sig figs

- hope this helps!

what is the sum of 3x4

Answers

Answer:

12

Step-by-step explanation:

3 4's

3+3+3+3=12

Hope that helps :)

The sum of this question is 12

Compute Z, corresponding to P28 for standard normal curve. 5. Random variable X is normally distributed with mean 36 and standard deviation 3. Find the 80th percentile.

Answers

The 80th percentile of the normal distribution with a mean of 36 and a standard deviation of 3 is approximately 38.52.

To compute Z corresponding to P28 for the standard normal curve, we need to find the Z-score that corresponds to a cumulative probability of 0.28. This can be done using a standard normal distribution table or a statistical software.

Using a standard normal distribution table, we can look up the cumulative probability closest to 0.28, which is 0.2794. The corresponding Z-score is approximately -0.59.

Therefore, Z corresponding to P28 for the standard normal curve is approximately -0.59.

Regarding the second part of your question, to find the 80th percentile of a normal distribution with a mean of 36 and a standard deviation of 3, we can use the Z-table or a statistical software.

The 80th percentile corresponds to a cumulative probability of 0.80. Using the Z-table or a statistical software, we can find the Z-score that corresponds to a cumulative probability of 0.80, which is approximately 0.84.

To find the actual value, we can use the formula:

Value = Mean + (Z-score * Standard Deviation)

Plugging in the values:

Value = 36 + (0.84 * 3) = 38.52

Therefore, the 80th percentile of the normal distribution with a mean of 36 and a standard deviation of 3 is approximately 38.52.

For more questions on standard deviation

https://brainly.com/question/475676

#SPJ8

"
Use the non- linear shooting method with accuracy 10^-1 (stop at 2nd iteration if this accuracy is not attained earlier) to solve the boundary-value problem: y"=-yy'+y, and 1<=x<=2, y(1)=1/2,y(2) =1/3. use h =0.5.Compare your results with actual solution : y(x) =1/ (x+1).

Answers

The non-linear shooting method with an accuracy of [tex]10^{-1}[/tex] was applied to solve the boundary-value problem y" = -yy' + y. The results were compared with the actual solution y(x) = 1/(x+1).

To solve the given boundary-value problem using the shooting method, we consider the problem as an initial-value problem by introducing an initial condition for y'(1).

Then, an iterative process is performed to find the appropriate value of y'(1) that satisfies the second boundary condition at x = 2.

Starting with an initial guess for y'(1), say y'(1) = a, we integrate the differential equation y" = -yy' + y numerically over the interval 1 <= x <= 2 using a step size of h = 0.5.

The numerical integration can be done using methods such as the Runge-Kutta method.

At each iteration, we compare the computed value of y(2) with the desired boundary condition y(2) = 1/3. If the accuracy of [tex]10^{-1}[/tex] is not attained after the second iteration, the process is stopped.

If the accuracy is achieved, the solution is considered as the actual solution.By comparing the obtained numerical solution with the actual solution y(x) = 1/(x+1), we can evaluate the accuracy of the non-linear shooting method.

The difference between the two solutions can be analyzed to assess the effectiveness of the method in solving the given boundary-value problem.

Learn more about boundary here:

brainly.com/question/26232363

#SPJ11

Please help I need help and please explain

Answers

Answer:

click c

Step-by-step explanation:

and its right

Help with my mathhh!

Answers

Answer:

Step-by-step explanation:

d

Answer:

<BOF

Step-by-step explanation:

sum of both these angles is 180 so C is the answer.

Which situation CANNOT be represented by this equation? 6−7=29 6 x - 7 = 29 CLEAR CHECK Joel earns $7 $ 7 per hour to mow his aunt's lawn. If he spends $6 $ 6 on gas, how many hours, x , will he need to mow to have $29 $ 29 left? Joel mows his aunt's lawn for $6 $ 6 per hour. If he spends $7 $ 7 on gas, how many hours, x , will he need to spend mowing the lawn to have $29 $ 29 left? Joel takes 6 6 hours to mow his aunt's lawn. If he spends $7 $ 7 on gas, how much does he get paid per hour, x , to mow the lawn and still have $29 $ 29 left?

Answers

Answer:

Joel earns $7 per hour to mow his aunt's lawn. If he spends $6 on gas, how many hours, x , will he need to mow to have $ 29 left?

Step-by-step explanation:

The given equation is :

6x - 7 = 29

It is given that total hours be = x

Joel earns $7  per hour, so the total earning of Joel in x hours is = $ 7x

He spends $6 on gas, so = 7x - 6

He is left with $ 29 after spending on gas.

Therefore, the equation becomes :

7x - 6 = 29

Clearly, it can be seen that in this case, the given situation cannot be represented by the given equation, i.e. 6x - 7 = 29.

PLSSS HELPPPPP

On the following number line, point C represents the integer -1. Identify the integer that each of the other letters represent.



A:

B:

D:

E:

Answers

Answer:  B is 0  A is 1 D 2 E 3

Step-by-step explanation:

Answer:

a: 1

b: 0

d:2

e:3

Step-by-step explanation:

Since you already know that C is -1, you can add or subtract along the numberline by ones to get the values of each letter

NEED THIS DONE ASAP


If P(x) = -2(1 - x)2 +5, what is the value of
P(-3)?

Answers

Answer:

P(-3) = -1

Step-by-step explanation:

So just substitute the value of x in the equation:

Value of x is (-3)

So:

P(x) = -2(1 - x)2 +5

P(-3) = -2(1-(-3)) 2 + 5

P(-3) = -2(4) 2 + 5

P(-3) = -8 + 2 + 5

P(-3) = -6 + 5

P(-3) = -1

Diddy Corp. Stock has a beta of 1.2, the current risk-free rate is 6 percent, and the expected return on the market is 14.50 percent. What is Diddy's cost of equity?

Answers

Answer: 16.2%

Step-by-step explanation:

You can find the cost of equity using the Capital Asset Pricing Model (CAPM).

Cost of equity = Risk free rate + Beta * (Expected return on market - Risk free rate)

= 6% + 1.2 * (14.50 - 6%)

= 6% + 10.2%

= 16.2%

Please help me with the question

Answers

90 degrees and right angle

90 degrees and now can u help me with my history work

Explain, using an example, why you need to multiply when converting from a larger unit to a smaller unit

Answers

Answer:

This is often called scaling in math, used in ratios, fraction, percent's etc.

Step-by-step explanation:

Answer:

You can use similar processes when converting from smaller to larger units. When converting a larger unit to a smaller one, you multiply; when you convert a smaller unit to a larger one, you divide. Here is an example..

Find the measure of each angles:

Answers

Answer:

10. 123º

13. 65º

Step-by-step explanation:

For number 10, a straight line is equal to 180º so all we have to do is subtract 57 (the number that we're given) from 180

For number 13, vertical angles are always congruent (the same) so we know that it's 65º

Given a quaternion with rotation of 90° about the x-axis and route point (1,0,1)

Find the following:

a. Scalar part
b. i, j, k components
c. Px, Py, Pz

Answers

Given the quaternion with rotation of 90° about the x-axis and route point (1,0,1), we have to find the scalar part, i, j, k components, Px, Py, Pz.

To find the scalar part, we need to use the formula: Scalar part = cos(θ/2)Where θ is the angle of rotation, which is 90° in this case. Scalar part = cos(90°/2) = cos(45°) = 0.7071To find the i, j, k components, we use the formula: qi = sin(θ/2) * ai where ai is the unit vector in the axis of rotation. i-component = sin(90°/2) * 1 = 1j-component = 0k-component = 0Therefore, the quaternion is (0.7071, 1i, 0j, 0k)To find Px, Py, Pz, we rotate the point (1,0,1) by the given quaternion using the formula: P' = qpq-1where q is the given quaternion, and P' is the new point.

Let's first find the inverse of the quaternion.q-1 = (0.7071, -1i, 0j, 0k) (Since the scalar part remains the same, only the vector part gets negated)Now, let's substitute the values and simplify: P' = (0.7071 + 1i)(1 + 0j + 0k)(0.7071 - 1i) = (0.7071 + 1i)(0.7071 - 1i) = 1 - 0.7071iTherefore, the new point is (1, 0, -0.7071)Hence, Px = 1, Py = 0, and Pz = -0.7071.

Know more about scalar part:

https://brainly.com/question/28518744

#SPJ11

Determine the value of A

Answers

Answer:

A = 161°

Step-by-step explanation:

125° + 36° = 161°

hope this helps

find the surface area of the part of the cone z=sqrt(x^2 y^2) that lies between the plane y=x and the cylinder y=x^2

Answers

The surface area of the part of the cone z = sqrt(x^2 + y^2) that lies between the plane y = x and the cylinder y = x^2 is sqrt(2)/6.

To find the surface area of the part of the cone z = sqrt(x^2 + y^2) that lies between the plane y = x and the cylinder y = x^2, we can use a double integral to integrate the surface area element dS over the region of interest.

First, we need to parameterize the surface in terms of two variables (u, v) such that the surface is defined by x = f(u,v), y = g(u,v), and z = h(u,v). We can use cylindrical coordinates, with x = r cos(theta), y = r sin(theta), and z = sqrt(x^2 + y^2) = r. Then, the cone is given by r = h(u,v) = u, and the region bounded by y = x and y = x^2 is given by u^2 <= v <= u.

Next, we need to compute the partial derivatives of f, g, and h with respect to u and v:

f_u = cos(theta)

f_v = -u sin(theta)

g_u = sin(theta)

g_v = u cos(theta)

h_u = 1

h_v = 0

Then, the surface area element dS can be computed using the formula:

dS = sqrt(1 + (h_u)^2 + (h_v)^2) du dv

Substituting in the partial derivatives and simplifying, we get:

dS = sqrt(2) du dv

Finally, we can set up the double integral over the region of interest and integrate dS:

surface area = ∫∫ dS = ∫[0,1]∫[u^2,u] sqrt(2) dv du

Evaluating this integral using the limits of integration gives us:

surface area = ∫[0,1] sqrt(2) (u - u^2) du

= sqrt(2) (1/2 - 1/3)

= sqrt(2)/6

Therefore, the surface area of the part of the cone z = sqrt(x^2 + y^2) that lies between the plane y = x and the cylinder y = x^2 is sqrt(2)/6.

Learn more about surface area here:

https://brainly.com/question/29298005

#SPJ11

What is the percent of 0.875?

Answers

Answer:

87.5%

Step-by-step explanation:

Just move the decimal place twice to the right.

Answer:

87.5%

Step-by-step explanation:

0.875× 100= 87.5%

Find X. Give your answer in the simplest form.

Answers

ok so this is a 30-60-90 triangle

x is 20 and y is 20 root 3

im sorry my phone is being weird which is why the writing turned out like that but if u have any questions lmk

What is the value of 9x^2 + 13x – 15, if x = 2

Answers

Step-by-step explanation:

Given,

[tex]9 {x}^{2} + 13x - 15[/tex]

and

[tex]x = 2[/tex]

Substitute x = 2 into expression.

[tex]9 {x}^{2} + 13x - 15 = 9( {2}^{2} ) + 13(2) - 15 \\ = 9(4) + 26 - 15 \\ = 36 + 26 - 15 \\ = 62 - 15 \\ = 47[/tex]

Answer:

9(2)×2+13(2)-15

18×2+26-15

36+11

47 is your answer ☺️☺️☺️

We will flip a balanced coin 3 times and for each toss, record whether we get a Head or a Tail. Write all possible outcomes of this experiment to find the probability that we get exactly 2 heads. 3/8 2/3 1/8 1/3

Answers

The probability of getting exactly 2 heads when flipping a balanced coin 3 times is 3/8.

When flipping a coin, each flip has 2 possible outcomes: Head (H) or Tail (T). Since we are flipping the coin 3 times, the total number of possible outcomes is 2 × 2 × 2 = 8. To find the probability of a specific outcome, we divide the number of favorable outcomes by the total number of possible outcomes. In this case, the favorable outcomes are the 3 outcomes with exactly 2 heads, and thus the probability is 3/8.

Learn more about probability here:

https://brainly.com/question/30034780

#SPJ11

Which is NOT true?
A 11 = 11
B 11 = 18 - 7
C 11 + 5 = 15 + 11
D 11 + 3 = 6 + 8

Answers

Answer:

C

Step-by-step explanation:

11+5=15

15+11=26

15 does not equal 26

Answer:C 11 + 5 = 15 + 11

Step-by-step explanation:

This is the only one that is false because the left side of the equation is equal to 16 and the right side of the equation is equal to 26

a large population is bi modal samples of sixe 40 are drawn in a sampling distribution

Answers

The given statement mentions a large population that exhibits a bimodal distribution. Bimodal distribution means that the data has two distinct peaks or modes.

Additionally, it states that samples of size 40 are drawn from this population, resulting in a sampling distribution.

A sampling distribution refers to the distribution of a statistic, such as the mean or proportion, calculated from multiple samples drawn from the same population. In this case, samples of size 40 are drawn, which means that each sample consists of 40 observations from the population.

The statement does not provide specific details about the purpose or objective of analyzing the sampling distribution. However, studying the sampling distribution can provide valuable insights into the behavior and properties of the population. It allows researchers to make inferences about the population parameters based on the statistics calculated from the samples.

To know more about sampling distribution click here: brainly.com/question/31465269

#SPJ11

Raymond takes a 28-inch by 21-inch rectangle of plywood and uses a table saw to cut from one corner of the piece of plywood to the diagonally opposite corner. Now Raymond has two equally sized triangles of plywood. What is the perimeter of each triangle?

Answers

Step-by-step explanation:

just to fit characters ...........

Find the value of x in the triangle shown below. ​

Answers

why dont u use a calculator
Yes you should use a calculator
Other Questions
Explain the three categories of biodiversity. ill mark brainly How can publicizing your participation in exercise increase yourLEMA credibility with the public? In order to remain viable, not-for-profit organizations need to focus on profits even more than the public service they provide.a. trueb. false A variable needs to be eliminated to solve the system of equations below. Choose thecorrect first step 5 + = 417x - 6y = -43 Should we allow concealed carrying of weapons? why? Scenario:You are interested in exploring your career options to make sure that you have the right skill set for your chosen field and that the future in the field looks promising. I'm studying Business Management and my career should be related to my degree.One of the best places to search for this information is the latest Occupational Outlook Handbook, which is compiled by the U.S. Bureau of Labor Statistics.Locate the Occupational Outlook Handbook A-Z Index online. Within the Index, search for your desired occupation. If you can not find your exact future occupation, select the closest match.Your Task:Write an informal memo report to your instructor that describes your career. In the report, summarize the information you found in the Handbook about the following:nature of the workworking conditionsnecessary qualificationsoutlook for the career.Also, summarize the relevant information about typical salary ranges for this occupation. You will need to read thoroughly and click-through links to be sure that you have all the information necessary.Be sure to add an appropriate introductory paragraph that describes the purpose of the report, and conclude with a brief paragraph that summarizes what you learned from this investigation. For instance, after investigation, do you still feel this career is a good fit for you? State your reasoning.Use effective formatting to make your report clear, including charts or tables when appropriate.Cite all information responsibly and appropriately.Be sure that you include all topics listed in the instructions above. I would recommend using headings and subheadings to make this information easy to locate. If I can't tell you included one of the topics (i.e., I can't find it easily), the report hasn't met the specifications listed. Determine whether the sentence below is complex or compound-complexThe Israel Museum, which is found in West Jerusalem, has artifacts dating back to prehistoric man, and it keeps them on display for visitors.a.complexb.compound-complexPlease select the best answer from the choices providedAB Please help me with the question Which of the following are components of an effective HR function?A. Singular focus on efficiencyB. Building capabilities to execute strategiesC. Dictating talent profiles based on identified selection toolsD. Considering practices individually and picking the most effective individual tactics from all possible systems Yea its D) Determining an Association of VariablesA group of adults were surveyed about whether they prefer to carpool or take the bus to work.Relative Frequency Table by ColumnA 3-column table with 3 rows. Column 1 has entries men, women, total. Column 2 is labeled carpool with entries 51 percent, 49 percent, 100 percent. Column 3 is labeled bus with entries 49 percent, 51 percent, 100 percent.Which conclusions can be drawn from the results of the survey? Check all that apply.A person in the group who is a man is more likely to carpool.A person in the group who is a woman is more likely to carpool.A person in the group who is a man is more likely to take the bus.There is no association between commuting preferences and gender.excited Give two benifets to the supermarket maneger and two benifets of customers of using barcods on all items Which of the following is not part of the Stockholders Equitysection of a balance sheet?Select one:a. Treasury Stockb. Common Stockc. Retained Earningsd. Patents and Copyrights Why is the issue of Northern Ireland a contentious part of Brexit?A. Northern Ireland is not part of Brexit and instead will have to reunify with Ireland.B. As a result of Brexit, Northern Ireland is no longer allowed to tradw with the rest of Great Britain (England, Wales, and Scotland).C. Northern Ireland was not allowed to vote in the Brexit referendum.D. Reinstating passport and customs inspections between Northern Ireland and Ireland may undermine the peace accord between the two.Before European currencies were linked in 1979,A. countries would competitively devalue their currencies to capture exports.B. countries had no independent monetary policy.C. countries informally tied their exchange rates to each other.D. most currencies were independently pegged to the pound or the dollar. below. TheyRead the conversation between Myrna and Nadiasome worker's Rightsarediscussinginfactory anymoremeyouNadia: Myrna, are you still on to leave?Myrna. I am not workingNadia: Haveyou resigned from your work?Myrna: No, I was dismissed. The momanagerwantedto report after a month of my delivery. I simplycan't becausemy baby is weakNadia: But have valid reason to continueyourmaternity leave your bed baby has a serioustot health condition And under the labor CodePemale employeeslworkers are given 60 days needmaternity leave after giving birthMyrna: I have reminded themofthat. I even beggedthem not to dismiss me.Vadia: But they can not just fine you while youron leave You have been working in that for 11 Y t, qu dices? Escribir - Hablar 1. Qu tipo de tiendas y servicios hay en el centro de tu comunidad? A qu hora se abren? A qu hora se cierran? Se cierran antes o despus de las seis de la tarde! 2. Cuando tu familia compra equipo deportivo, dnde lo compra? Y dnde compra cosaa como jabn o pasta dental? 3. Te gusta caminar? A qu lugares puedes ir a pie fcilmente en tu comunidad? 4. Qu haces para ganar dinero? Te gusta cuidar a los nios? Por qu? What is the value of 3 14 increased by 2 16? Please help Sammy buys light bulbs in pack of 8 for $20. The shipping cost is $10 regardless of the number of packs bought. Billy has only $120 to spend. Find the radius of convergence, R, of the series. (-7) Vn 00 ya n=1 R= Find the interval, I, of convergence of the series. Read the stanza from "Jabberwocky.""And, has thou slain the Jabberwock?Come to my arms, my beamish boy!O frabjous day! Callooh! Callay!"He chortled in his joy.Based on the context, what is the meaning of the nonsense language in the third line?The language is used to indicate expressions of happiness.The langauge is used as an expression of concern.The langauge indicates that the speaker is frightened by the Jabberwock.The language is used to paint a picture for the reader. Find the internal rates of return on a cash flow with deposit amounts of Ao 120, A = 290, and withdrawal amounts of Bo = 240, B = 20, B : - A - at times t = 0, t= 1, t = 2, respectively. 40, 10,